Prove Same Cardinality (1,3) and [1,4]

  • Thread starter knowLittle
  • Start date
  • Tags
    Cardinality
You can then use a bijection between (1, 3) and [1, 4] from your post to finish the proof.In summary, to prove that the open interval (1,3) and the closed interval [1,4] have the same cardinality, we must prove bijection. By using the function y(x) = 3x/2 - 1/2, we can map (1,3) onto (1,4) in a one-to-one and onto manner. Additionally, we can use this function to map irrational numbers to irrational numbers and vice-versa. For rational numbers in (1,3), we can order them as x1, x2, etc. and then
  • #1
knowLittle
312
3

Homework Statement


Prove that the open interval (1,3) and the closed interval [1,4] have the same cardinality.


Homework Equations





The Attempt at a Solution


I have to prove bijection.
The injective part is obvious.

Say, A =(1,3) and B =[1,4]
f: A → B
f(x) = x
It's injective.

I have problems with g: B → A
g(x) = 1+ ... ?

Help. I plan to use Cantor, Schroder, and Bernstein's theorem.
 
Physics news on Phys.org
  • #2
How would you do an injection from [1,4] to [2,3], say?
 
  • #3
haruspex said:
How would you do an injection from [1,4] to [2,3], say?

The mapping would be ## h(x) = 1 + \frac{x}{2} ## does it for x =4 from Domain, but not for x =1 from Range.

Is there an easier way to get this.

I appreciate your help LCKurtz, but it's more difficult for me in that way.
 
Last edited:
  • #4
knowLittle said:

Homework Statement


Prove that the open interval (1,3) and the closed interval [1,4] have the same cardinality.


Homework Equations





The Attempt at a Solution


I have to prove bijection.
The injective part is obvious.

Say, A =(1,3) and B =[1,4]
f: A → B
f(x) = x
It's injective.

I have problems with g: B → A
g(x) = 1+ ... ?

Help. I plan to use Cantor, Schroder, and Bernstein's theorem.

Here's an idea you can try. If I wanted to map surjectively [0,1] to (0,1], I could use the identity map except there's no place for 0 to map to. So I could try something like this:$$
0\to \frac 1 2,~\frac 1 2\to \frac 1 3,~\frac 1 3\to \frac 1 4 ...$$and all other values of x to themselves. See if you can convince yourself that that is a surjection and then apply the idea to your problem.
 
  • #5
knowLittle said:
The mapping would be ## h(x) = 1 + \frac{x}{2} ## does it for x =4 from Domain, but not for x =1 from Range.

Is there an easier way to get this.

I appreciate your help LCKurtz, but it's more difficult for me in that way.

You could use the straight line between the points (1,2) and (4,3) to answer Haruspex's question. That isn't what you did. What is that equation? It's a similar problem to yours. But you will have problems at the missing end points for your problem. You are going to eventually have to think about my suggestion.
 
  • #6
LCKurtz said:
Here's an idea you can try. If I wanted to map surjectively [0,1] to (0,1], I could use the identity map except there's no place for 0 to map to. So I could try something like this:$$
0\to \frac 1 2,~\frac 1 2\to \frac 1 3,~\frac 1 3\to \frac 1 4 ...$$and all other values of x to themselves. See if you can convince yourself that that is a surjection and then apply the idea to your problem.

Ok, then I will try your way. Note that I need to map from a closed interval to an open interval. Your suggestion doesn't show that. Should it matter?
 
  • #7
knowLittle said:
Note that I need to map from a closed interval to an open interval.
More precisely, you need to map a closed interval into an open interval. There is no requirement to map onto the open interval. If you get your head around that you'll see the question is really very easy.
 
  • #8
haruspex said:
More precisely, you need to map a closed interval into an open interval. There is no requirement to map onto the open interval. If you get your head around that you'll see the question is really very easy.
I understand that I only require the function to be injective, but it still needs to be defined.
So, everything coming from my domain [1,4] , should be at least defined in (1,3), it can't be a number outside of (1,3).

However, I still don't see a function that would do it :/


g(x) = 1+ x/3 ?? It can really be anything like this, right? This g satisfies injection.
 
  • #9
knowLittle said:
g(x) = 1+ x/3 ?? It can really be anything like this, right? This g satisfies injection.
Looks ok to me.
 
  • Like
Likes 1 person
  • #10
knowLittle said:
Ok, then I will try your way. Note that I need to map from a closed interval to an open interval. Your suggestion doesn't show that. Should it matter?

My suggestion used a half open interval and showed you how to handle that end point. Also, for the record, I was trying to show you how to write down a specific surjection and avoid using the Shroeder-Bernstein theorem. That's a great theorem but, in my opinion, overkill for this problem.
 
  • Like
Likes 1 person
  • #11
Thanks LCKurtz and haruspex, I really appreciate your time.

LCKurtz, I would love to learn your way, but I am super pressed off time. Finals in a few hours :/
I would try the other method in the future for sure. Thanks!
 
  • #12
The function y(x)= 3x/2- 1/2 maps (1, 3) onto (1, 4). Further it maps irrational numbers to irrational numbers and vice-versa. So use that to map all irrational x to y(x). The rational numbers in (1, 3) can be ordered- call then [itex]x_1[/itex], [itex]x_2[/itex], etc. Use that to map all rational numbers in (1, 3) to all rational numbers in [1, 4].
 

Related to Prove Same Cardinality (1,3) and [1,4]

What does "Prove Same Cardinality (1,3) and [1,4]" mean?

The phrase "Prove Same Cardinality (1,3) and [1,4]" is a mathematical statement that is asking whether the two sets, (1,3) and [1,4], have the same number of elements.

How do you prove that two sets have the same cardinality?

To prove that two sets have the same cardinality, you must show that there exists a one-to-one correspondence between the elements of the two sets. This means that for every element in one set, there is exactly one corresponding element in the other set.

What is the cardinality of the set (1,3)?

The cardinality of a set is the number of elements it contains. In this case, the set (1,3) contains two elements, so its cardinality is 2.

Why does the set (1,3) have the same cardinality as the set [1,4]?

Although the notation may be different, the sets (1,3) and [1,4] contain the same number of elements. Both sets contain the numbers 1, 2, and 3, so they have the same cardinality of 3.

Can you provide an example of a one-to-one correspondence between (1,3) and [1,4]?

Yes, a one-to-one correspondence between (1,3) and [1,4] could be: 1 --> 1, 2 --> 2, and 3 --> 4. This shows that each element in (1,3) corresponds to exactly one element in [1,4], and vice versa.

Similar threads

  • Precalculus Mathematics Homework Help
Replies
4
Views
2K
  • Set Theory, Logic, Probability, Statistics
Replies
19
Views
2K
  • Precalculus Mathematics Homework Help
Replies
12
Views
2K
  • Precalculus Mathematics Homework Help
Replies
3
Views
1K
  • Set Theory, Logic, Probability, Statistics
Replies
9
Views
1K
  • Precalculus Mathematics Homework Help
Replies
7
Views
1K
  • Precalculus Mathematics Homework Help
Replies
3
Views
1K
  • Math Proof Training and Practice
Replies
5
Views
1K
  • Calculus and Beyond Homework Help
Replies
2
Views
987
  • Precalculus Mathematics Homework Help
Replies
9
Views
2K
Back
Top